On a busy airport, an aeroplane lands after an average of 15 minutes. Based on Poisson distribution, what is the probability that in a 15-minute interval, 3 or more aeroplanes will land?

Answers

Answer 1

The probability that 3 or more airplanes will land in a 15-minute interval is approximately 0.08.

We can use the Poisson distribution formula to solve this problem:

P(X >= 3) = 1 - P(X < 3)

where X is the number of airplanes that land in a 15-minute interval and P(X < 3) is the probability that 0, 1, or 2 airplanes land.

The average number of airplanes that land in 15 minutes is 1, so λ = 1.

Using the Poisson formula, we get:

P(X < 3) = e^(-λ) * (λ^0 / 0! + λ^1 / 1! + λ^2 / 2!)

P(X < 3) = e^(-1) * (1/1 + 1/1 + 1/2)

P(X < 3) = 0.9197

Therefore, the probability of 3 or more airplanes landing in a 15-minute interval is:

P(X >= 3) = 1 - P(X < 3) = 1 - 0.9197 = 0.0803 or approximately 0.08.

Learn more about probability here :

https://brainly.com/question/30034780

#SPJ11


Related Questions

what is the minimum neutral demand load (in kw) for 12 apartments, each containing an 8-kw range

Answers

Minimum neutral demand load is approximately 23.04 kw.To determine the minimum neutral demand load for 12 apartments, each containing an 8-kw range, we need to add up the individual demand loads of each apartment and divide by three (since the neutral carries only the unbalanced load).

The demand load for an 8-kw range is typically calculated at 5.76 kw (72% of 8 kw). Therefore, the total demand load for 12 apartments would be 12 x 5.76 kw = 69.12 kw. Dividing this by three gives us a minimum neutral demand load of approximately 23.04 kw. It's important to note that this calculation assumes all ranges are being used simultaneously, which may not always be the case.

To know more about  Minimum neutral demand load visit:

brainly.com/question/31064989

#SPJ11

Two identical tubes, each closed at one end, have a fundamental frequency of 349 Hz at 25.0$^\circ$CC. The air temperature is increased to 31.0$^\circ$CC in one tube. If the two pipes are now sounded together, what beat frequency results? noise power if the output signal is 10 W?

Answers

When the two tubes are sounded together after one has been heated to 31.0°C, a beat frequency of 4 Hz will result.

We must first comprehend how the basic frequency is impacted by the change in temperature in order to respond to your query.

The speed of sound increases along with an increase in air temperature. The following equation can be used to determine the speed of sound in air at a temperature T (in Celsius):


v = 331.4 * sqrt(1 + T/273.15)

Let's calculate the speed of sound for both temperatures:

v1 = 331.4 * sqrt(1 + 25/273.15) ≈ 346.74 m/s (at 25.0°C)
v2 = 331.4 * sqrt(1 + 31/273.15) ≈ 349.67 m/s (at 31.0°C)

Now that the tube's temperature has raised, we need to determine its new fundamental frequency. Since the frequency and sound speed are directly related, we may establish the following ratio:


f1 / f2 = v1 / v2

Solving for f2, we have:

f2 = f1 * (v2 / v1)

f2 = 349 Hz * (349.67 / 346.74) ≈ 353 Hz

Now that we have the new fundamental frequency for the heated tube (353 Hz), we can find the beat frequency by taking the difference between the two frequencies:

Beat frequency = |f2 - f1| = |353 Hz - 349 Hz| = 4 Hz
To know more about the fundamental frequency, click here;

https://brainly.com/question/29264927

#SPJ11

Consider light from a helium-neon laser ( \(\lambda= 632.8\) nanometers) striking a pinhole with a diameter of 0.375 mm.At what angleto the normal would the first dark ring be observed?

Answers

The first dark ring would be observed at an angle of approximately 25.8 degrees to the normal. The first dark ring in a diffraction pattern is observed when the path difference between the light waves from the top and bottom of the pinhole is equal to one wavelength.

The angle at which this occurs is given by :- sinθ = λ/D

Where θ is the angle to the first dark ring, λ is the wavelength of the light,

D is the diameter of the pinhole.

Substituting the values given:

sinθ = (632.8 nm) / (0.375 mm)

sinθ = 0.423

θ = sin⁻¹(0.423) = 25.8 degrees

To know more about diffraction refer here :-

https://brainly.com/question/12290582#

#SPJ11

the 5-kgkg collar is initially at rest at position 1. a constant 100-nn force is applied to the string, causing the collar to slide up the smooth vertical bar. What is the velocity of the collar when it reaches position 2? Express your answer with the appropriate units.

Answers

The velocity of the collar when it reaches position 2 is 8.94 m/s.

To find the velocity of the collar when it reaches position 2, we need to use the principles of force and velocity. According to Newton's second law, the force applied to an object is equal to its mass multiplied by its acceleration. Therefore, we can find the acceleration of the collar by dividing the applied force by its mass.
Acceleration = Force / Mass = 100 N / 5 kg = 20 m/s²
Next, we can use the equation of motion to find the velocity of the collar at position 2.
v² = u² + 2as
Where, v is the final velocity, u is the initial velocity (which is zero), a is the acceleration, and s is the distance traveled.
We know that the collar is moving up a smooth vertical bar, which means there is no frictional force, and hence, the distance traveled (s) is simply the vertical height between position 1 and position 2. Let's assume that the distance is 2 meters.
v² = 0 + 2 x 20 x 2
v² = 80
v = √80
v = 8.94 m/s
Therefore, the velocity of the collar when it reaches position 2 is 8.94 m/s.

To know more about velocity visit: https://brainly.com/question/19979064

#SPJ11

A 60 cm valve is designed to control the flow in a pipeline. A 1/3 scale model of the valve will be tested with water in the laboratory at full scale. If the flow rate of the prototype is going to be 0.5 m3/s, what flow rate should be established in the laboratory test to have dynamic similarity?
Also, if it is found that the coefficient
The model's CP pressure is 1.07, what will be the corresponding CP on the full scale valve? The properties
relevant to the oil fluid are SG=0.82 and μ = 3x10 -3 N s/m2 .

Answers

The flow rate in the laboratory test should be 0.02 m3/s to achieve dynamic similarity and corresponding CP on the full scale valve is 4.99.

To achieve dynamic similarity between the prototype and the model valve, the following equation can be used:
(Q_model / Q_prototype) = (D_model / D_prototype)^2 * (CP_model / CP_prototype)^0.5
Where:
Q = flow rate
D = diameter
CP = pressure coefficient
Substituting the given values:
Q_prototype = 0.5 m3/s
D_prototype = 60 cm = 0.6 m
D_model = 0.6 m * (1/3) = 0.2 m
CP_model = 1.07 (given)
Solving for Q_model:
(Q_model / 0.5 m3/s) = (0.2 m / 0.6 m)^2 * (1.07 / CP_prototype)^0.5
Q_model = 0.02 m3/s
Therefore, the flow rate in the laboratory test should be 0.02 m3/s to achieve dynamic similarity.
To find the corresponding CP on the full scale valve:
CP_prototype = CP_model * (SG_model / SG_prototype) * (V_model / V_prototype)^2
Where:
SG = specific gravity
V = velocity
Substituting the given values:
SG_prototype = 0.82 (given)
SG_model = 1 (water)
V_prototype = Q_prototype / (pi/4 * D_prototype^2) = 0.5 m/s
V_model = Q_model / (pi/4 * D_model^2) = 3.18 m/s
Solving for CP_prototype:
CP_prototype = 1.07 * (1 / 0.82) * (3.18 m/s / 0.5 m/s)^2
CP_prototype = 4.99
Therefore, the corresponding CP on the full scale valve is 4.99.

To know more about pressure visit:

brainly.com/question/29341536

#SPJ11

From greatest to least, rank the accelerations of the boxes. Rank from greatest to least. To rank items as equivalent, overlap them. Reset Help 10 N<-- 10 kg -->15 N 5 N<-- 5 kg -->10 N 15 N<-- 20 kg -->10 N 15 N<-- 5 kg -->5NGreatest Least

Answers

To rank the accelerations of the boxes from greatest to least, we need to apply Newton's second law, which states that the acceleration of an object is directly proportional to the force applied to it and inversely proportional to its mass. That is, a = F/m.

First, let's calculate the acceleration of each box. For the 10 kg box with a 10 N force, a = 10 N / 10 kg = 1 m/s^2. For the 5 kg box with a 5 N force, a = 5 N / 5 kg = 1 m/s^2. For the 20 kg box with a 15 N force, a = 15 N / 20 kg = 0.75 m/s^2. Finally, for the 5 kg box with a 15 N force, a = 15 N / 5 kg = 3 m/s^2.

Therefore, the accelerations from greatest to least are: 5 kg box with 15 N force (3 m/s^2), 10 kg box with 10 N force (1 m/s^2) and 5 kg box with 5 N force (1 m/s^2), and 20 kg box with 15 N force (0.75 m/s^2).

In summary, the 5 kg box with a 15 N force has the greatest acceleration, followed by the 10 kg box with a 10 N force and the 5 kg box with a 5 N force, and finally, the 20 kg box with a 15 N force has the least acceleration.

Learn more about Acceleration :

https://brainly.com/question/460763

#SPJ11

You are in the back of a pickup truck on a warm summer day and you have just finished eating an apple. The core is in your hand and you notice the truck is just passing an open dumpster 7. 0 m due west of you. The truck is going 30. 0 km/h due north and you can throw that core at 60. 0 km/h. In what direction should you throw it to put it in the dumpster, and how long will it take it to reach its destination?

Answers

To put the apple core in the dumpster, you should throw it at an angle of approximately 23.6 degrees north of west. It will take approximately 0.067 seconds for the apple core to reach the dumpster.

To determine the angle at which you should throw the apple core, we need to analyze the velocities of both the truck and the throw. The truck is moving due north at 30.0 km/h, and you can throw the apple core at 60.0 km/h. We can break down the velocities into their horizontal and vertical components.

The horizontal component of the truck's velocity does not affect the apple core's trajectory since it is moving perpendicular to the throw. However, the vertical component of the truck's velocity needs to be considered. By using the concept of relative velocity, we can subtract the vertical component of the truck's velocity from the vertical component of the throw's velocity to achieve the desired direction.

To calculate the time it takes for the apple core to reach the dumpster, we can use the horizontal distance between you and the dumpster (7.0 m) and the horizontal component of the apple core's velocity. Since the time is the same for both the horizontal and vertical components, we can use the horizontal component of the velocity to calculate the time.

By applying the relevant equations and calculations, the angle should be approximately 23.6 degrees north of west, and the time it takes for the apple core to reach the dumpster is approximately 0.067 seconds.

Learn more about velocity here

https://brainly.com/question/29801541

#SPJ11

The jet engine has angular acceleration of -2.5 rad/s2. Which one of the following statements is correct concerning this situation? 1. The direction of the angular acceleration is counterclockwise. 2. The direction of the angular velocity must be clockwise. 3. The angular velocity must be decreasing as time passes. 4. If the angular velocity is clockwise, then its magnitude must increase as time passes. 5. If the angular velocity is counterclockwise, then its magnitude must increase as time passes.

Answers

Answer:

The direction of the angular acceleration is counterclockwise.

Explanation:

Angular acceleration is a vector quantity and has both magnitude and direction. The negative sign indicates that the angular acceleration is in the opposite direction to the initial angular velocity.

In this case, the negative angular acceleration of -2.5 rad/s2 indicates that the engine is slowing down, which means that the angular acceleration is in the opposite direction to the angular velocity, and hence it must be counterclockwise.

Statement 2 is incorrect because the direction of the angular velocity is not specified, and it can be either clockwise or counterclockwise.

Statement 3 is correct because the negative angular acceleration implies that the angular velocity is decreasing as time passes.

Statement 4 is incorrect because the direction of the angular velocity is not specified, and the magnitude of the angular velocity may increase or decrease depending on its direction.

Statement 5 is also incorrect for the same reason as statement 4.

To know more about angular acceleration refer here

brainly.com/question/29428475#

#SPJ11

urrent results in a magnetic moment that interacts with the magnetic field of the magnet. will the interaction tend to increase or to decrease the angular speed of the coil?

Answers

When a current flows through a coil, it generates a magnetic moment that interacts with the magnetic field of a nearby magnet.

This interaction between the magnetic moment and the magnetic field creates a torque on the coil. According to Lenz's Law, this torque will act in a direction to oppose the change in magnetic flux. As a result, the interaction will tend to decrease the angular speed of the coil.

Faraday's law states that when there is a change in the magnetic flux through a coil, an electromotive force (EMF) is induced, which in turn leads to the generation of an electric current. This principle forms the basis of many electrical devices, such as generators and transformers.

Lenz's law, on the other hand, provides information about the direction of the induced current and its associated magnetic field. According to Lenz's law, the induced current will always flow in such a way as to oppose the change in the magnetic flux that caused it.

This opposition creates a magnetic moment that interacts with the magnetic field of the nearby magnet, resulting in a torque on the coil.

The torque generated by this interaction tends to resist the change in motion of the coil. If the coil is initially rotating, the torque will act to decrease its angular speed.

Similarly, if an external force tries to rotate the coil, the torque will resist that motion. This opposition to changes in motion is a fundamental principle of electromagnetic interactions and is known as Lenz's law.

To learn more about coil, refer below:

https://brainly.com/question/27961451

#SPJ11

The position of a mass oscillating on a spring is given by x = ( 3.6 cm)cos[2pi t/(0.67s)].
A. What is the period of this motion?
T=? s
B. What is the first time the mass is at the position x = 0?
t=? s

Answers

The position of a mass oscillating on a spring is given by x = ( 3.6 cm)cos[2pi t/(0.67s)] the period of this motion is 0.671 s.

A. The period of the motion is given by T = 2π/ω, where ω is the angular frequency. The angular frequency is given by ω = 2π/T, so we can rearrange this equation to find T = 2π/ω.

In this case, we are given x = (3.6 cm)cos[2πt/(0.67 s)], so the angular frequency is ω = 2π/(0.67 s) = 9.39 s^(-1).

Therefore, the period is T = 2π/ω = 2π/(9.39 s^(-1)) ≈ 0.671 s.

B. We are given that x = (3.6 cm)cos[2πt/(0.67 s)], and we want to find the first time the mass is at the position x = 0. This occurs when the argument of the cosine function is equal to π/2, 3π/2, 5π/2, etc.

In other words, we want to solve the equation (2πt)/(0.67 s) = π/2 + nπ, where n is an integer. Rearranging this equation, we get t = (0.67 s/2π)(π/2 + nπ) = (0.335 s) + (0.335 s)n.

The first time the mass is at the position x = 0 corresponds to n = 0, so we get t = 0.335 s. Therefore, the first time the mass is at the position x = 0 is t ≈ 0.335 s.

To learn more about mass oscillating refer here:

https://brainly.com/question/30905479#

#SPJ11

light of wavelength shiens on the metals lithium, iron, an dmercury, which have work functions of 2.3 ev, 3.9 ev, and 4.5 ev, respectively

Answers

The minimum energy of the incident light needed to eject electrons from lithium, iron, and mercury are 2.3 eV, 3.9 eV, and 4.5 eV, respectively.

When light is shone on a metal surface, the photons of the light can transfer their energy to electrons in the metal. If the energy of the photons is greater than the work function of the metal (i.e., the minimum energy required to remove an electron from the metal), then the electrons can be ejected from the metal surface. This process is called the photoelectric effect.

In this scenario, the wavelength of the incident light is not specified, so we cannot determine the energy of the photons. However, we do know the work function of each metal. Therefore, we can determine the minimum energy of the incident light needed to eject electrons from each metal. For lithium, the minimum energy is 2.3 eV; for iron, it is 3.9 eV; and for mercury, it is 4.5 eV.

Learn more about wavelength here:

https://brainly.com/question/31143857

#SPJ11

A hiker stands at the edge of a clear alpine lake that is 4.10 m deep. (Use 1.33 for the (a) What is the apparent depth of the lake? m (b) Returning in the summer, the hiker finds the lake surface 1.10m lower than before. What is the apparent depth of the lake now?

Answers

(a) The apparent depth of the lake is 3.08 meters, (b)The apparent depth of the lake is now 2.26 meters, the refraction of light as it passes from the air to the water,

The apparent depth of the lake is the depth that the hiker perceives when looking into the water. This depth is affected by the refraction of light as it passes from the air to the water, and it can be calculated using the formula : apparent depth = real depth / refractive index

where the refractive index is the ratio of the speed of light in air to the speed of light in water, which is approximately 1.33.

Substituting the given values, we get:

apparent depth = 4.10 m / 1.33

apparent depth = 3.08 m

(b)

new real depth = 4.10 m - 1.10 m

new real depth = 3.00 m

Using the same formula as before, we can calculate the new apparent depth:

apparent depth = new real depth / refractive index

apparent depth = 3.00 m / 1.33

apparent depth = 2.26 m

The lower water level has reduced the apparent depth of the lake as seen by the hiker.

To know more about refraction refer here :-

https://brainly.com/question/29692290#

#SPJ11

A 75 kg cyclist turns a corner with a radius of 40 m at a speed of 20 m/s. What is the magnitude of the cyclist's centripetal force

Answers

When the cyclist turns the corner with a radius of 40 m at a speed of 20 m/s, the magnitude of the centripetal force required to keep the cyclist in the circular path is 750 N.

Centripetal Force: Centripetal force is the force that keeps an object moving in a curved path. It acts towards the center of the circular path and is required to maintain circular motion.

Formula for Centripetal Force: The formula to calculate the centripetal force is:

F = (m * v^2) / r

where F is the centripetal force, m is the mass of the object, v is the velocity, and r is the radius of the circular path.

Given Values: In this scenario, the mass of the cyclist is 75 kg, the speed is 20 m/s, and the radius of the corner is 40 m.

Calculating the Centripetal Force: Substituting the given values into the formula, we have:

F = (75 kg * (20 m/s)^2) / 40 m

F = (75 kg * 400 m^2/s^2) / 40 m

F = 750 N

Therefore, the magnitude of the cyclist's centripetal force is 750 N.

For more such questions on centripetal force, click on:

https://brainly.com/question/20905151

#SPJ8

Let’s explore the superposition of two waves, y1 and y2, where:
Y1= sin(πx − 2πt) and Y2= sin(πx÷2 + 2πt)
Write down the physical properties that you can determine for both waves, y1 and y2. Graph these two waves by hand based on your deduction of the properties. For simplicity, remove time-dependent behavior from our consideration and take t = 0.
Now, let’s superimpose the two waves. It makes the most sense to explore the superposition graphically. Draw a second graph in your notebook showing y1 + y2. Think about the best way to go about doing this and explain why you chose the method that you used.

Answers

Physical properties of waves Y1 and Y2: amplitude=1, wavelengths (λ1=2, λ2=4), frequencies (f1=1/2, f2=1/4), phases (φ1=-2π, φ2=2π); Superposition graph of y1 + y2 accurately represented by creating a table, calculating the sum of y1 and y2 for each x value, and plotting the points.

What are the physical properties of waves Y1 and Y2, and how can the superposition graph y1 + y2 be accurately represented?

For the waves Y1 and Y2, we can determine the following physical properties:

Amplitude (A): The amplitude of a wave represents the maximum displacement from the equilibrium position. In this case, both waves have an amplitude of 1.

Wavelength (λ): The wavelength is the distance between two consecutive points in the wave that are in phase. Since both waves have a sin function, we can determine the wavelength by examining the coefficient of x in each wave's argument. For Y1, the wavelength is given by λ1 = 2π/π = 2. For Y2, the wavelength is λ2 = 2π/(π/2) = 4.

Frequency (f): The frequency is the number of oscillations per unit time. In this case, the frequency can be calculated as the reciprocal of the wavelength. For Y1, the frequency is f1 = 1/λ1 = 1/2. For Y2, the frequency is f2 = 1/λ2 = 1/4.

Phase (φ): The phase of a wave indicates its position relative to a reference point. In Y1, the phase is determined by the coefficient of t, which is -2π. In Y2, the phase is given by 2π.

Now, let's graph these two waves at t = 0:

For Y1: y1 = sin(πx)

For Y2: y2 = sin(πx/2)

To graphically represent the superposition y1 + y2, we need to add the values of y1 and y2 for each corresponding x. The best way to do this is by creating a table with values of x and calculating the sum of y1 and y2 at each x value. This will allow us to plot the points and draw the graph accurately.

Let's create the table and graph for the superposition y1 + y2:

x    |   y1 = sin(πx)   |   y2 = sin(πx/2)   |   y1 + y2

---------------------------------------------------------

-2   |       0          |        0           |    0

-1   |       0          |        0           |    0

0    |       0          |        0           |    0

1    |       0          |        1           |    1

2    |       0          |        0           |    0

By calculating the sum of y1 and y2 at each x value, we can see that the superposition y1 + y2 is 0 for x = -2, -1, 0, and 2, while it is 1 for x = 1. This information allows us to plot the points on the graph and draw a curve connecting them.

The chosen method of creating a table and calculating the sum of y1 and y2 is the most accurate and reliable way to graphically represent the superposition. It ensures that we consider all possible values of x and obtain the correct sum of the two waves at each x value. This approach eliminates errors that could occur if we attempted to visually estimate the shape of the superposition graph without performing the calculations explicitly.

Learn more about   Superposition graph

brainly.com/question/12493909

#SPJ11

A 6.5 kg cat is near the edge of a 7 m diameter merry-go-round in a playground. A man pushes and accelerates the merry-go-round from rest at a uniform rate of 0.91 rad/s2 until the angular velocity reaches 5.5 rad/s. How long did it take for the merry go round to get up to this speed? t = S Over what angle did the merry-go-round rotate during its acceleration? 0 rad How many rotations did the merry-go-round make at this point? rotations

Answers

To calculate the time it took for the merry-go-round to reach a speed of 5.5 rad/s, we can use the formula t = v_f - v_i / a.

Plugging in the values, we get:
t = (5.5 rad/s - 0 rad/s) / 0.91 rad/s^2
t = 6.04 s
Finally, to calculate the number of velocity the merry-go-round made at this point, we can use the formula: rotations = θ / 2π
where θ is the angle in radians. Plugging in the value we just found, we get: rotations = 16.6 rad / 2π
rotations = 2.65 rotations

Therefore, the merry-go-round made approximately 2.65 rotations during its acceleration. Using the formula for rotational motion, ω² = ω₀² + 2αθ, where ω is the final angular velocity, ω₀ is the initial angular velocity, α is the angular acceleration, and θ is the angle over which the acceleration.

To know more about velocity visit :-

https://brainly.com/question/31780003

#SPJ11

When researchers implanted electrodes into a person's hippocampus, they found cells sensitive to what? A. Color B. Temperature C. Location D. Rhyming.

Answers

When researchers implanted electrodes into a person's hippocampus, they found cells sensitive to location. The hippocampus is responsible for spatial navigation and memory, so it makes sense that it would have cells that are sensitive to location.

This discovery has important implications for our understanding of how the brain works and how we form memories of the world around us. It also has potential applications in the development of new treatments for disorders such as Alzheimer's disease, which is characterized by a breakdown in memory function. By understanding how the hippocampus works at the cellular level, researchers may be able to develop new therapies to help people with memory impairments.


When researchers implanted electrodes into a person's hippocampus, they found cells sensitive to "C. Location." These cells are called place cells, and they play a crucial role in spatial navigation and memory formation. Place cells fire in response to specific locations within an environment, creating a cognitive map for navigation. This discovery has significantly contributed to our understanding of how the brain processes and stores information about our surroundings, ultimately helping us navigate through the world.

To know more about electrodes visit:

https://brainly.com/question/17060277

#SPJ11

let a_2a 2 be recessive, let qq be the frequency of the a_2a 2 allele, and let there be selection against the a_2a_2a 2 a 2 genotype. in that case, q=1q=1 is a/an

Answers

Answer:If the a2a2 genotype experiences selection against it, then its frequency will decrease in subsequent generations. Assuming the selection is strong enough, the genotype may be eliminated from the population altogether.

In this scenario, q represents the frequency of the a2 allele, and q=1 would mean that the a1 allele has been fixed in the population. This implies that there are no more a2 alleles left in the gene pool, and all individuals are homozygous for the a1 allele.

Therefore, q=1 is an indication of complete fixation of the a1 allele in the population, and the a2 allele has been lost due to selection against the a2a2 genotype.

Learn more about population genetics and allele frequencies.

https://brainly.com/question/16043998?referrer=searchResults

#SPJ11

a 1260-kg car moves at 21.0 m/s. how much work net must be done on the car to increase its speed to 35.0 m/s?

Answers

The initial speed of the car is 21.0 m/s and the final speed is 35.0 m/s. The change in speed is:

Δv = vf - vi = 35.0 m/s - 21.0 m/s = 14.0 m/s

The mass of the car is 1260 kg. We can use the kinetic energy formula to find the initial and final kinetic energies of the car:

Ki = (1/2)mv^2 = (1/2)(1260 kg)(21.0 m/s)^2 = 284,715 J

Kf = (1/2)mv^2 = (1/2)(1260 kg)(35.0 m/s)^2 = 765,450 J

The net work done on the car is equal to the change in kinetic energy:

Wnet = Kf - Ki = 765,450 J - 284,715 J = 480,735 J

Therefore, the net work that must be done on the car to increase its speed from 21.0 m/s to 35.0 m/s is 480,735 J.

To know more about speed refer here

https://brainly.com/question/28224010#

#SPJ11

2.37 a lossless transmission line is terminated in a short circuit. how long (in wavelengths) should the line be for it to appear as an open circuit at its input terminals?

Answers

To determine the length of a lossless transmission line that appears as an open circuit at its input terminals when terminated in a short circuit, we need to consider the standing waves that are generated along the line. When a lossless transmission line is terminated in a short circuit, a standing wave is created with a voltage maximum at the load end and a current maximum at the input end.

To achieve an open circuit at the input terminals, we need to locate a point along the line where the voltage is a minimum. This occurs at a distance of λ/4 from the input terminals, where λ is the wavelength of the signal on the line. At this point, the current is at a maximum and the voltage is at a minimum, effectively creating an open circuit. Therefore, the length of the line that would appear as an open circuit at its input terminals is equal to λ/4. We can calculate the wavelength λ using the formula λ = v/f, where v is the velocity of the signal on the transmission line and f is the frequency of the signal.

Learn more about frequency here:

https://brainly.com/question/30783512

#SPJ11

true/false. question content area using a naive forecasting method, the forecast for next week’s sales volume equals

Answers

Using a naive forecasting method, the forecast for next week’s sales volume equals. The given statement is true because naive forecasting is a straightforward method that assumes the future will resemble the past

It relies on the most recent data point (in this case, the current week's sales volume) as the best predictor for future values (next week's sales volume). This method is simple, easy to understand, and can be applied to various content areas.

However, it's essential to note that naive forecasting may not be the most accurate or reliable method for all situations, as it doesn't consider factors such as trends, seasonality, or external influences that may impact sales volume. Despite its limitations, naive forecasting can be useful in specific scenarios where data is limited, patterns are relatively stable, and when used as a baseline for comparison with more sophisticated forecasting techniques. So therefore the given statement is true because naive forecasting is a straightforward method that assumes the future will resemble the past, so the forecast for next week’s sales volume equals.

Learn more about naive forecasting here:

https://brainly.com/question/31580569

#SPJ11

what is the longest-wavelength em radiation (in nm) that can eject a photoelectron from osmium, given that the binding energy is 5.93 ev? nm is this in the visible range? yes no

Answers

The longest-wavelength EM radiation that can eject a photoelectron from osmium is 209 nm. This is not in the visible range, as the visible range for humans is approximately 400-700 nm.

The energy of a photon is given by the equation E = hc/λ, where E is energy, h is Planck's constant, c is the speed of light, and λ is wavelength. To eject a photoelectron, the energy of the photon must be greater than or equal to the binding energy of the electron. The binding energy for osmium is given as 5.93 eV.

Using the equation E = hc/λ and converting electron volts to joules, we can solve for the maximum wavelength as follows:

5.93 eV * 1.602 x 10^-19 J/eV = 9.51 x 10^-19 J (binding energy)

h = 6.626 x 10^-34 J s (Planck's constant)

c = 2.998 x 10^8 m/s (speed of light)

λ = hc/E = (6.626 x 10^-34 J s)(2.998 x 10^8 m/s)/(9.51 x 10^-19 J) = 209 nm.

To know more about wavelength, refer here:

https://brainly.com/question/13047641#

#SPJ11




10 POINTS!


The heater is designed to work from a 3. 6V supply it has a power rating of 4. 5W at this voltage.


By considering the current in the heater, calculate the resistance of component X when there is the correct potential difference across the heater.

Answers

The resistance of component X is 2.88 Ω when there is the correct potential difference across the heater.

Given that the heater is designed to work from a 3.6V supply and has a power rating of 4.5W at this voltage. We know that the power of the heater is 4.5W and voltage across the heater is 3.6V.The relationship between power, voltage and current is given by the formula:

Power = Current * Voltage .So, we can calculate the current in the heater as: I = \frac{P }{VI }= \frac{4.5 }{ 3.6I} = 1.25A .

Using Ohm's law, we know that: V = IR ,Where V is the voltage across the heater, I is the current in the heater and R is the resistance of the heater. Rearranging the above equation, we get:

R = \frac{V }{ IR} =\frac{ 3.6 }{1.25R} = 2.88 Ω

Therefore, the resistance of component X is 2.88 Ω when there is the correct potential difference across the heater. Note: Power is the rate at which work is done. It is expressed in Watts (W). Resistance is the opposition offered by a material to the flow of electric current through it. It is measured in Ohms (Ω).

Learn more about potential difference Refer: https://brainly.com/question/23899758

#SPJ11

The average speed of a perfume vapor molecule at room temperature is about 300 m/s, but you find the speed at which the scent travels across the room is much less than that. Explain why this is so

Answers

The average speed of a perfume vapor molecule is about 300 m/s at room temperature. However, the scent travels across the room at a much slower speed due to the random motion of the molecules, diffusion, and interactions with air molecules.

These factors slow down the overall movement of the scent and cause it to spread gradually. While individual perfume vapor molecules may have an average speed of 300 m/s, the scent as a whole does not move at that speed across the room. The movement of scent is primarily driven by diffusion, which is the random motion of molecules from an area of high concentration to an area of low concentration. As the perfume molecules diffuse, they collide with air molecules, other perfume molecules, and objects in the room, causing them to change direction and slow down. These interactions and collisions result in a gradual and slower spread of the scent throughout the room, rather than a rapid propagation at the individual molecule's average speed.The average speed of a perfume vapor molecule is about 300 m/s at room temperature. However, the scent travels across the room at a much slower speed due to the random motion of the molecules, diffusion, and interactions with air molecules.

learn more about speed here:

https://brainly.com/question/32298217

#SPJ11

A pot of boiling water with a temperature of 100°C is set in a room with a temperature of 20°C. The temperature T of the water after x hours is given by T(x) = 20 + 80 e *. (a) Estimate the temperature of the water after 2 hours. (b) How long did it take the water to cool to 30°C? After 2 hours, the tempertaure of the water will be approximately (Type an integer or decimal rounded to one decimal place as needed.) The water will cool to 30°C in about hour(s). (Type an integer or decimal rounded to two decimal places as needed.)

Answers

If a  pot of boiling water with a temperature of 100°C is set in a room with a temperature of 20°C. The temperature T of the water after x hours is given by T(x) = 20 + 80 e *(a) After 2 hours, the temperature of the water will be approximately 56.6°C (rounded to one decimal place).
(b)the water will never cool to 30°C,

To find out how long it takes for the water to cool to 30°C, we can set T(x) = 30 and solve for x:

30 = 20 + 80e⁻ⁿˣ

Subtracting 20 from both sides:

10 = 80e⁻ⁿˣ

Dividing by 80:

1/8 = e⁻ⁿˣ

Taking the natural logarithm of both sides:

ln(1/8) = -nx

Solving for x:

x = ln(1/8) / -n

We know that the initial temperature of the water is 100°C, so we can use that to find k:

100 = 20 + 80e⁻ⁿ⁽⁰⁾

80 = 80

So n= 0.

Plugging that into the equation for x:

x = ln(1/8) / 0

This is undefined, but we know that the water will cool to 30°C eventually, so we can take the limit as T(x) approaches 30:

lim x-> infinity ln(1/8) / -n = infinity

This means that the water will never cool to 30°C, because it would take an infinite amount of time.

To learn more about temperature https://brainly.com/question/26866637?cb=1683177743053

#SPJ11

How much energy is required to raise the air temperature from 68°f to 72°f, neglecting heat transfer to the walls, floor, and ceiling?

Answers

Approximately 2.32 x 10⁶ J of energy is required to raise the air temperature from 68°F to 72°F.

The amount of energy required to raise the air temperature from 68°F to 72°F depends on the mass of air being heated, specific heat of air and the temperature difference.

Using the formula Q = mcΔT, where Q is the energy required, m is the mass of air being heated, c is the specific heat of air, and ΔT is the change in temperature, we can calculate the energy required to raise the air temperature from 68°F to 72°F.

Assuming a room with dimensions of 10 ft x 10 ft x 8 ft, and a density of air at standard temperature and pressure (STP) of 1.225 kg/m³, we can calculate the mass of air in the room to be approximately 1041 kg.

The specific heat of air at constant pressure is 1005 J/(kg*K).

Converting the temperature difference to Kelvin, we have ΔT = 4°F = 2.22°C = 2.22 K.

Thus, the energy required to raise the air temperature from 68°F to 72°F is:

Q = mcΔT = (1041 kg)(1005 J/(kg*K))(2.22 K) = 2.32 x 10⁶ J

Therefore, approximately 2.32 x 10⁶ J of energy is required to raise the air temperature from 68°F to 72°F.

To know more about air temperature refer here:

https://brainly.com/question/27754428#

#SPJ11

the magnetic moment of a hydrogen nucleus is roughly 2.82×10−26j/t . what would be the resonant frequency f in a 5.00 t magnetic field?

Answers

The resonant frequency (f) can be calculated using the formula f = µB/h, where µ is the magnetic moment, B is the magnetic field, and h is Planck's constant.

In order to determine the resonant frequency (f) of a hydrogen nucleus in a 5.00 T magnetic field, we can use the formula f = µB/h.

Here, µ is the magnetic moment (2.82×[tex]10^(-^2^6)[/tex] J/T), B is the magnetic field strength (5.00 T), and h is Planck's constant (6.626×[tex]10^(^-^3^4^)[/tex] Js).

Plugging in these values, we get f = (2.82×[tex]10^(^-^2^6[/tex]) J/T)(5.00 T) / (6.626×[tex]10^(^-^3^4^)[/tex] Js). After calculating, the resonant frequency is approximately 2.13× [tex]10^8[/tex] Hz or 213 MHz, which is the frequency needed for resonance in the given magnetic field.

For more such questions on frequency, click on:

https://brainly.com/question/28995449

#SPJ11

The resonant frequency (f) of a hydrogen nucleus in a 5.00 T magnetic field is approximately 7.16 × 10^(-27) Hz.To calculate the resonant frequency (f) of a hydrogen nucleus in a 5.00 T magnetic field, we can use the formula:

f = γB / 2π

where f is the resonant frequency, γ is the gyromagnetic ratio, B is the magnetic field strength, and π is the mathematical constant pi (approximately 3.14159).

Given the magnetic moment (μ) of a hydrogen nucleus is roughly 2.82 × 10^(-26) J/T, we can calculate the gyromagnetic ratio (γ) using the formula:

γ = μ / I

where I is the nuclear spin quantum number. For a hydrogen nucleus, I = 1/2.

Thus, γ = (2.82 × 10^(-26) J/T) / (1/2) = 5.64 × 10^(-26) J/T.

Now, we can plug this value of γ and the given magnetic field strength (B) of 5.00 T into the resonant frequency formula:

f = (5.64 × 10^(-26) J/T × 5.00 T) / 2π

f ≈ 4.50 × 10^(-26) J / 6.283

f ≈ 7.16 × 10^(-27) Hz

Therefore, the resonant frequency (f) of a hydrogen nucleus in a 5.00 T magnetic field is approximately 7.16 × 10^(-27) Hz.

learn more about resonant frequency here: brainly.com/question/13040523

#SPJ11

The breaking strength X[kg] of a certain type of plastic block is normally distributed with a mean of 1250kg and a standard deviation of 5.5kg. What is the maximum load such that we can expect no more than 55% of the blocks to break?

Answers

The maximum load such that we can expect no more than 55% of the blocks to break is 1250.691 kg.

To find the maximum load such that no more than 55% of the blocks break, we need to use the mean, standard deviation, and percentile information of the normal distribution. Here are the steps:

1. Convert the percentage (55%) to a decimal: 0.55.

2. Look up the z-score corresponding to 0.55 in a standard normal table or use a calculator. The z-score is approximately 0.1257.

3. Use the formula: X = μ + (z * σ), where X is the maximum load, μ is the mean, z is the z-score, and σ is the standard deviation.

Applying the formula:

X = 1250 + (0.1257 * 5.5)

X ≈ 1250 + 0.691

X ≈ 1250.691 kg

So, the maximum load such that we can expect no more than 55% of the blocks to break is approximately 1250.691 kg.

Learn more about standard deviations here,

https://brainly.com/question/31616931

#SPJ11

a coul of area a = 0.85m2 is rotatin with angular speed w = 290 rad/s with magnetic field. The coil has N 350 turns.

Answers

The coil has N 350 turns and therefore the induced EMF in the coil is equal to -89125 times the magnetic field.

When this coil rotates within a magnetic field, it generates an electromotive force (EMF) according to Faraday's law of electromagnetic induction. The formula to calculate the maximum EMF is:

EMF_max = N * A * B * ω * sin(θ)

In this formula, B represents the magnetic field strength and θ is the angle between the magnetic field and the normal to the coil's plane.

The magnetic field causes an induced EMF in the coil, given by the equation:

EMF = -N(wB)A

where N is the number of turns in the coil, w is the angular speed of the coil, B is the magnetic field, and A is the area of the coil. Plugging in the given values, we get:

EMF = -(350)(290)(B)(0.85) = -89125B

So the induced EMF in the coil is equal to -89125 times the magnetic field.

More on induced EMF: https://brainly.com/question/31102118

#SPJ11

Consider a planet of mass m that has a circular orbit of radius r around a star of mass M >> m. The planet's Hill radius ry is defined such that at this distance from the planet toward the star, the forces on an orbiting test mass will be in balance. a. At such a distance rh from the planet, and r - rh from the star, write out the combined acceleration gtot from the star's gravity and the planet's gravity, as well as the centrifugal acceleration from orbiting the star with the same period as the planet. b. Now set this &tot = 0, and solve for ry in terms of m, M, and r, under the approximations m

Answers

a. The combined acceleration gtot at distance rh from the planet in a circular orbit around the star with radius r is given by gtot = -(GM/r^2)rh + (Gm/r^2)(r - rh) + (v^2/rh), where G is the gravitational constant, M is the mass of the star, m is the mass of the planet, and v is the orbital velocity of the planet.

b. Setting gtot = 0 and solving for ry, the Hill radius is approximately given by ry = r[(m/3M)^(1/3)]. This approximation assumes that m << M and that the orbit of the planet is circular. The Hill radius is the maximum distance from the planet where its gravity dominates over the star's gravity and where objects can be stably bound to the planet.

To calculate the combined acceleration, we must consider the gravitational forces of both the star and the planet on an orbiting test mass at distance rh from the planet.

The centrifugal acceleration is also included as it must be balanced by the gravitational forces. Setting gtot to zero and solving for ry involves algebraic manipulation and the use of the approximation that m << M and the orbit is circular.

For more questions like Acceleration click the link below:

https://brainly.com/question/12550364

#SPJ11

If a point charge is located at the center of a cube and the electric flux through one face of the cubeis 5.0 Nm2/C, what is the total flux leaving the cube?
A) 1 Nm2/C
B) 20 Nm2/C
C) 5.0 Nm2/C
D) 30 Nm2/C
E) 25 Nm2/C

Answers

30 Nm2/C is the total flux leaving the cube. Option D) is correct .

The total electric flux leaving the cube is given by Gauss's law, which states that the total flux through any closed surface is equal to the charge enclosed divided by the electric constant, ε₀. Since the point charge is located at the center of the cube, the charge enclosed by the cube is equal to the charge of the point charge.

The total flux leaving the cube can be found by multiplying the flux through one face by the total number of faces. A cube has 6 faces, so the total flux leaving the cube is:

Total flux = (flux through one face) x (number of faces)
Total flux = 5.0 Nm2/C x 6
Total flux = 30 Nm2/C

Therefore, If a point charge is located at the center of a cube and the electric flux through one face of the cubeis 5.0 Nm2/C then total flux leaving the cube is (D) 30 Nm2/C.

To know more about Flux refer here :

https://brainly.com/question/14751407

#SPJ11

Other Questions
Commonly occurring and well-defined business activities such as scheduling employees work hours would be what category of organizational decision?. A plant raised in a sterile environment shows symptoms of a viral infection. Which of the following statements is the best explanation of this?a. The viral infection was acquired by the environment.b. The viral infection was inherited from a parent.c. The viral infection was developed by a mutation in the plant's genes.d. The viral infection was picked up by horizontal transmission. What are three modern day parallels to the book 1984 part 2 with quotes? how are (s),(r)-stilbene dibromide and (r),(s)-stilbene dibromide related to one another? Turn back to Figure 15.1, which lists the prices of various Microsoft options. Use the data in the figure to calculate the payoff and the profits for investments in each of the following November 2019 expiration options, assuming that the stock price on the expiration date is $140.call option, x = 140 put option, x = 140 A pendulum is made by tying a 410 g ball to a 49.0 cm long string. The pendulum is pulled 21.0 degrees to one side, then released.(a) What is the ball's speed at the lowest point of its trajectory?(b) To what angle does the pendulum swing on the other side? Antitrust policy seeks to prevent or eliminate which of the following practices?A. firing of workersB. monopolists practices of underproducing and overchargingC. pollution of riversD. monopolistic competitionE. discrimination in the labor market cardiac muscle cells obtain energy almost exclusively through which, if any, of these works rely on stroboscopic motion to create a sense of movement? the assignment goal is to develop a program in java that keeps track of all your travels incorporating inheritance and polymorphism structures. you are interested in creating a mouse with suppressed expression of a particular gene of interest. which of these would likely be your method of choice to artificially regulate the gene? 9. John and Colin have chocolate milkin pyramid-shaped containers likethe ones shown.John-6 cm8 cm8 cmColin10 cm5 cmWho has more chocolate milk?How much more?A Colin; 1 cm moreB They have the same amount.C John; 2 cm moreD Colin; 2 cm more Leila is choosing a main course and a dessert in a restaurant. There are 5 main courses and 7 desserts to choose from. a) work out the number of combinations available to Leila. Explain the main differences between weak sustainability (neoclassical) and strong sustainability (ecological), and why these matter for government policy on sustainable development. List the two requirements for development to be sustainable FILL IN THE BLANK by going __________ for some or all server resources, you can reduce your power usage and increase uptime. which condition is recognized as one that will affect the ventilation component of the ventilation/perfusion ratio? Need helpFor childhood education class:-How do you make structure and activity for 1st grade math class? Question 1 For this question, you'll be using Statkey In Statkey, go to "Test for Single Proportion" in the Randomization Hypothesis Tests. You'll be adding the data in by clicking "Edit Data" and inputting the relevant information. Situation: A sample of 50 U.S. adults were asked if they wear a helmet when they go for a bike ride. 35 of the adults in the sample said "yes" they wore a helmet. Does this provide evidence to suggest that a majority of U.S adults wear a helmet when they ride their bike? I need help on how to do the chart and statkey and how it should look, as well as how to solve my problem and how I can show the work for it. The total cost, in dollars, to produce bins of cat food is given by C(x)=9x+13650.The revenue function, in dollars, is R(x) = -2x + 469xFind the profit function.P(x) =At what quantity is the smallest break-even point?Select an answer Monochromatic light from a helium laser (? = 632.8 nm) is incident normally on a diffraction grating containing 6.00 x 103 lines/cm. Find the angles at which one would observe the first order maximum, the second order maximum, and so forth.